Using Laplace Transform , solve the following differential equation {y}text{}-{4}{y}={e}^{{-{3}{t}}},{y}{left({0}right)}={0},{y}'{left({0}right)}={2}

Tahmid Knox

Tahmid Knox

Answered question

2020-12-01

Using Laplace Transform , solve the following differential equation
y4y=e3t,y(0)=0,y(0)=2
a) 1420e2t530e2t930e6t
b) 1120e2t5120e2t420e3t
c) 1415e2t510e2t920e3t
d) 1420e2t+520e2t920e3t

Answer & Explanation

broliY

broliY

Skilled2020-12-02Added 97 answers

Step 1
As per guidelines, only typed answers are acceptable. so we will give only typed solution.
Given initial value problem is
y4y=e3t,y(0)=0,y(0)=2
Step 2
Applying Laplace transform on both sides
L{y4y}=L{e3t}
L{y}4L{y}=1s+3
s2L{y}sy(0)y(0)4L{y}=1s+3
(s24)L{y}2=1s+3
L{y}=1(s+3)(s24)+2s24
=(15(s+3)+120(s2)14(s+2))+(12(s+2)+12(s2))
=15(s+3)+1120(s2)34(s+2)
Taking inverse Laplace transform on both sides, we get
y=L1{15(s+3)+1120(s2)34(s+2)}
=L1{15(s+3)}+L1{1120(s2)}L1{34(s+2)}
=15e3t+1120e2t34e2t
Hence, solution is
y(t)=1120e2t34e2t+15e3t  or  y(t)=1120e2t1520e2t+420e3t
Comparing the answer with options, none of the option is correct.
The option given are all wrong.

Do you have a similar question?

Recalculate according to your conditions!

New Questions in Differential Equations

Ask your question.
Get an expert answer.

Let our experts help you. Answer in as fast as 15 minutes.

Didn't find what you were looking for?